A cylinder has a radius of 5 inches and a height of 8 inches. What is its volume rounded to the nearest tenth?

Answers

Answer 1

Answer:

630

Step-by-step explanation:

v= pi×radius squared×hight so its 3.14×5 sqared×8 628 and 628 rounded to the nearest tenth is 630 hope this help


Related Questions

[tex]\left\{\frac{1}{2^n}\:+\:\frac{\left(-1\right)^n}{n+1}\::\:n\:\:N\right\}[/tex]
Find max, min, sup and inf

Answers

The maximum value of the sequence is [tex]1/6[/tex], the minimum value is 0, the supremum is [tex]1/2[/tex], and the infimum is 1/6.

The given sequence is: [tex]{1/(2 ^ n) + ((- 1) ^ n)/(n + 1) / n * N}[/tex]

To find the maximum and minimum values of the sequence, we can start by taking the first few terms and looking for patterns:

When [tex]n = 1,[/tex] the sequence evaluates to: [tex]1/2 + (-1)^1 / 2 * 2 = 0[/tex]

When [tex]n = 2,[/tex] the sequence evaluates to: [tex]1/4 + (-1)^2 / 3 * 2 = 1/6[/tex]

When [tex]n = 3,[/tex] the sequence evaluates to: [tex]1/8 + (-1)^3 / 4 * 2 = 7/96[/tex]

When [tex]n = 4,[/tex] the sequence evaluates to: [tex]1/16 + (-1)^4 / 5 * 2 = 17/240[/tex]

It appears that the sequence oscillates between positive and negative values, with the negative values getting smaller as n increases.

Therefore, the minimum value of the sequence is at n = 1, where it evaluates to 0.

The maximum value occurs at n = 2, where it evaluates to 1/6.

To find the supremum and infimum of the sequence, we can start by considering the upper and lower bounds of each term separately.

The term [tex]1/(2 ^ n)[/tex] has a lower bound of 0 and an upper bound of 1.

The term [tex]((- 1) ^ n)/(n + 1) / n * N[/tex] has a lower bound of [tex]-1/4[/tex] and an upper bound of [tex]1/4[/tex].

Therefore, the supremum of the sequence occurs when [tex]n = 1[/tex], where the sequence evaluates to [tex]1/2[/tex].

The infimum of the sequence occurs when [tex]n = 2[/tex], where the sequence evaluates to [tex]1/6.[/tex]

In summary, the maximum value of the sequence is [tex]1/6[/tex], the minimum value is 0, the supremum is [tex]1/2[/tex], and the infimum is [tex]1/6.[/tex]

Learn more about maximum and minimum values here:

https://brainly.com/question/14316282

#SPJ1

Springfield will be opening a new high school in the fall. The number of underclassmen (9th and 10th graders) must fall between 600 and 700
(inclusive), the number of upperclassmen (11th and 12th graders) must fall between 500 and 600 (inclusive), and the number of students cannot
exceed 1200. Let a represent the number of underclassmen and let b represent the number of upperclassmen. Write a set of inequalities that
models the constraints on the composition of the student body.
number of underclassmen:
number of upperclassmen:
Total number of students:
:: 600 < a < 700
000
:: 600 ≤ a ≤ 700
:: 500 ≤ b ≤ 600
:: a + b ≤ 1200
:: 500 < b < 600
:: a + b > 1200
= a + b < 1200
:: a + b > 1200

Answers

The correct set of inequalities that model the constraints on the composition of the student body are:

600 ≤ a ≤ 700, 500 ≤ b ≤ 600 and a + b ≤ 1200

What is inequalities?

In mathematics, an inequality is a mathematical statement that indicates that two expressions are not equal. It is a statement that compares two values, usually using one of the following symbols: "<" (less than), ">" (greater than), "≤" (less than or equal to), or "≥" (greater than or equal to).

The correct set of inequalities that model the constraints on the composition of the student body are:

600 ≤ a ≤ 700 (the number of underclassmen must fall between 600 and 700, inclusive)

500 ≤ b ≤ 600 (the number of upperclassmen must fall between 500 and 600, inclusive)

a + b ≤ 1200 (the total number of students cannot exceed 1200)

Note that the inequalities 600 < a < 700 and 500 < b < 600 are not correct, as they do not take into account the inclusive limits of the ranges for the number of underclassmen and upperclassmen. Also, the inequality a + b > 1200 is not correct, as it contradicts the previous inequality a + b ≤ 1200.

To learn more about inequalities from the given link:

https://brainly.com/question/30231190

#SPJ1

Y=2x to the power of 2 plus 4x minus one

Answers

This is the answer to your problem.

Q3: Use the image to determine the direction and angle of rotation.

graph of triangle ABC in quadrant 4 and a second polygon A prime B prime C prime in quadrant 3

90° clockwise rotation
90° counterclockwise rotation
180° clockwise rotation
360° counterclockwise rotation

Answers

the direction and angle of rotation between the two polygons is 180° clockwise rotation.

How to solve the question?

Based on the given information, we can determine the direction and angle of rotation between the two polygons.

First, let's look at the initial positions of the polygons. The graph of triangle ABC is located in Quadrant 4, which means that it is in the bottom-right portion of the coordinate plane. On the other hand, the second polygon A'B'C' is located in Quadrant 3, which is in the bottom-left portion of the coordinate plane.

To find the direction and angle of rotation between the two polygons, we need to imagine rotating one polygon onto the other. We can see that the two polygons are mirror images of each other across the y-axis. Therefore, we can infer that there is a horizontal line of symmetry between the two polygons.

If we rotate polygon A'B'C' 180 degrees clockwise around the origin, it will overlap perfectly with triangle ABC. This is because a 180-degree rotation is equivalent to a half-turn or a flip, which is exactly what we need to make the two polygons overlap. Therefore, the answer is 180° clockwise rotation.

In summary, the direction and angle of rotation between the two polygons is 180° clockwise rotation.

To know more about polygon visit :-

https://brainly.com/question/26583264

#SPJ1

Determine how many integer solutions there are to

x₁ + x₂ + x3 + x₁ = 20, if
0≤x₁ < 3, 0≤ x₂ < 4, 0≤x3 <5, 0≤x4 < 6

Answers

Based on the information given, there are a total of 118 solutions.

How many possible solutions are there?

This is a problem of solving a Diophantine equation subject to some conditions. Let's introduce a new variable y4 = 20 - (x1 + x2 + x3 + x4). Then the problem can be restated as finding the number of solutions to:

x1 + x2 + x3 + y4 = 20

Subject to the following conditions:

0 ≤ x1 < 3

0 ≤ x2 < 4

0 ≤ x3 < 5

0 ≤ y4 < 6

We can solve this problem using the technique of generating functions. The generating function for each variable is:

(1 + x + x^2) for x1

(1 + x + x^2 + x^3) for x2

(1 + x + x^2 + x^3 + x^4) for x3

(1 + x + x^2 + x^3 + x^4 + x^5) for y4

The generating function for the equation is the product of the generating functions for each variable:

(1 + x + x^2)^3 (1 + x + x^2 + x^3 + x^4 + x^5)

We need to find the coefficient of x^20 in this generating function. We can use a computer algebra system or a spreadsheet program to expand the product and extract the coefficient. The result is: 1118

Learn more about solutions in https://brainly.com/question/30665317

#SPJ1

Answer: This problem involves finding the number of non-negative integer solutions to the equation x₁ + x₂ + x3 + x₁ = 20 subject to the given constraints. We can use the stars and bars method to solve this problem.

Suppose we have 20 stars representing the sum x₁ + x₂ + x3 + x₁. To separate these stars into four groups corresponding to x₁, x₂, x₃, and x₄, we need to place three bars. For example, if we have 20 stars and 3 bars arranged as follows:

**|**||

then the corresponding values of x₁, x₂, x₃, and x₄ are 2, 4, 6, and 8, respectively. Notice that the position of the bars determines the values of x₁, x₂, x₃, and x₄.

In general, the number of ways to place k identical objects (stars) into n distinct groups (corresponding to x₁, x₂, ..., xₙ-₁) using n-1 separators (bars) is given by the binomial coefficient (k+n-1) choose (n-1), which is denoted by C(k+n-1, n-1).

Thus, the number of non-negative integer solutions to the equation x₁ + x₂ + x3 + x₁ = 20 subject to the given constraints is:

C(20+4-1, 4-1) = C(23, 3) = 1771

However, this count includes solutions that violate the upper bounds on x₁, x₂, x₃, and x₄. To eliminate these solutions, we need to use the principle of inclusion-exclusion.

Let Aᵢ be the set of non-negative integer solutions to the equation x₁ + x₂ + x3 + x₁ = 20 subject to the given constraints, where xᵢ ≥ mᵢ for some integer mᵢ. Then, we want to find the cardinality of the set:

A = A₀ ∩ A₁ ∩ A₂ ∩ A₃

where A₀ is the set of all non-negative integer solutions to the equation x₁ + x₂ + x3 + x₁ = 20, and Aᵢ is the set of solutions that violate the upper bound on xᵢ.

To find the cardinality of A₀, we use the formula above and obtain:

C(20+4-1, 4-1) = 1771

To find the cardinality of Aᵢ, we subtract the number of solutions that violate the upper bound on xᵢ from the total count. For example, to find the cardinality of A₁, we subtract the number of solutions where x₂ ≥ 4 from the total count. To count the number of solutions where x₂ ≥ 4, we fix x₂ = 4 and then count the number of solutions to the equation x₁ + 4 + x₃ + x₄ = 20 subject to the constraints 0 ≤ x₁ < 3, 0 ≤ x₃ < 5, and 0 ≤ x₄ < 6. This count is given by:

C(20-4+3-1, 3-1) = C(18, 2) = 153

Similarly, we can find the cardinalities of A₂ and A₃ by fixing x₃ = 5 and x₄ = 6, respectively. Using the principle of inclusion-exclusion, we obtain:

|A| = |A₀| - |A

Step-by-step explanation:

Gabriella is 53 5/6
inches tall. Sheila is 1 1/3
inches shorter than Gabriella and Jane is 1 1/4
inches shorter than Sheila. How tall is Jane?

Answers

Jane is 51 1/4 inches.

Subtracting Sheila’s height from Gabriella’s = 52 1/2

52 1/2 minus Jane’s height of 1 1/4 = 51 1/4, or 51.25 inches

Find the length of the third side. If necessary, write in simplest radical form.
3√3 and 6

Answers

Answer: 3

Step-by-step explanation:

You would need to use the Pythagorean theorem to solve this equation. 6 is the hypotenuse and 3[tex]\sqrt{3}[/tex] is the longer leg.  

a^2+b^2=c^2

c= hypotenuse

6^2=3[tex]\sqrt{3}[/tex]^2 + a^2

36=27+a^2

36-27=9

[tex]\sqrt{9}[/tex] = 3

Find the value of x. Write your answer in simplest form.
76√2

Answers

The value of x which is the hypotenuse of the triangle is 107.48

How to find missing side of a right angle triangle using Pythagoras theorem

[tex]\dfrac{\text{Opposite}}{\text{Hypotenuse}}[/tex]

[tex]\text{Hyp = x}[/tex]

[tex]\text{opp} = 76\sqrt{2}[/tex]

[tex]\text{adj} = \text{x}[/tex]

substitute into the equation

[tex]\text{x}^2 = (76\sqrt{2})[/tex]

[tex]\text{x}^2 = 11552[/tex]

[tex]\text{x}^2 = \sqrt{11552}[/tex]

[tex]\text{x} = 107.48[/tex]

Learn more about Pythagoras theorem:

brainly.com/question/231802

A flagpole is 12 feet fall. Its shadow is
11 feet long. How far is it from the top of the flagpole to the end of its shadow?

Answers

Step-by-step explanation:

You are looking for the hypotenuse of a right triangle with legs of 12 and

 11 feet

Using Pthagorean theorem

hypot^2 = 12^2 + 11^2

hypot^2 = 265

hypot = sqrt (265) = 16.28 ft

NO LINKS!! URGENT HELP PLEASE!!
Select all that apply

b. Symmetric with respect to the x-axis

Answers

The ones that are symmetric with respect to the x-axis is:

y = -7x^2

Checking the symmetric for all equations

A function is symmetric with respect to the x-axis if replacing y with -y in the equation does not change the equation. In other words, if the graph of the function is the same when reflected across the x-axis.

y = -7x^2 is symmetric with respect to the x-axis, since replacing y with -y gives -(-y) = y and the equation remains the same.y = 6x² - 9 is not symmetric with respect to the x-axis, since replacing y with -y gives -(-y) = y, but the equation changes to -y = 6x² - 9, which is not the same as the original equation.x=1/4y^2 is not a function, since it does not pass the vertical line test and has multiple values of x for some values of y.y=x^3-1 is not symmetric with respect to the x-axis, since replacing y with -y gives -(-y) = y, but the equation changes to -y = x^3 - 1, which is not the same as the original equation.x=-y^2+9 is not symmetric with respect to the x-axis, since replacing y with -y gives -(-y) = y, but the equation changes to x = -(-y)^2 + 9, which is not the same as the original equation.y=sqrt(x) is not symmetric with respect to the x-axis, since replacing y with -y gives -(-y) = y, but the equation changes to -y = sqrt(x), which is not the same as the original equation.y=sqrt(x)-6 is not symmetric with respect to the x-axis, since replacing y with -y gives -(-y) = y, but the equation changes to -y = sqrt(x) - 6, which is not the same as the original equation.

Therefore, only the equation y = -7x^2 is symmetric with respect to the x-axis.

Learn more about symmetric here:

https://brainly.com/question/14405062

#SPJ1

A six sided dice is rolled. What is the probability of rolling a number greater than 2?

Answers

The probability of rolling a number greater than 2 is 2/3

Calculating the probability of rolling a number greater than 2?

From the question, we have the following parameters that can be used in our computation:

Rolling a number cube once

Using the above as a guide, we have the following:

Sample space,  S = {1, 2, 3, 4, 5, 6}

In the above sample space, we have

Outcomes greater than 2 = {3, 4, 5, 6}

So, we have

P(greater than 2) = n(Outcomes greater than 2)/n(Sample space)

Substitute the known values in the above equation, so, we have the following representation

P(greater than 2) = 4/6

When evaluated, we have

P(greater than 2) = 2/3

Hence, the probability is 2/3

Read more about probability at

https://brainly.com/question/24756209

#SPJ1

A quadratic function yields negative values between x = 2 and x = 6. Its minimum value is −2. What are the coordinates of the y-intercept? Enter your answer by filling in the boxes.

Answers

Answer:

Since the quadratic function has a minimum value at some point between x = 2 and x = 6, its graph is a downward-facing parabola.

Let's assume that the function is of form f(x) = ax^2 + bx + c, where a, b, and c are constants.

Since the minimum value of the function is −2, we know that the vertex of the parabola lies on the line y = -2. Also, we know that the x-coordinate of the vertex is the average of 2 and 6, which is 4.

Therefore, the equation of the parabola can be written as f(x) = a(x-4)^2 - 2.

Since the y-intercept is the value of y when x = 0, we can find it by plugging in x = 0 into the equation of the parabola:

f(0) = a(0-4)^2 - 2

f(0) = 16a - 2

We know that the function yields negative values between x = 2 and x = 6, so the parabola must intersect the y-axis below the x-axis. This means that the y-intercept is negative.

To find the y-intercept, we need to solve the equation 16a - 2 = 0, which gives us a = 1/8.

Therefore, the equation of the parabola is f(x) = (1/8)(x-4)^2 - 2.

Finally, we can find the y-intercept by plugging in x = 0:

f(0) = (1/8)(0-4)^2 - 2

f(0) = 8 - 2

f(0) = 6

So the coordinates of the y-intercept are (0, 6).

in the figure below lines m and n are parallel m2= 62 and m3=73

Answers

Answer: 135

Step-by-step explanation:

What angle(s) on the Unit Circle make this equation true?
-√3 csc(2θ) = 2

a. Using only the graph of the given equation on Desmos, what angle(s) on the Unit Circle make this equation true? You must include a detailed, labeled screenshot as your explanation or detailed, labeled drawing of the graph as you solution.

b. Even though Desmos found the angle(s) that make the equation true in part (a), you must now show why the angle(s) are true. Provide a clear, convincing argument why the angles you stated in part (a) are true without the use of Desmos in anvway.

Answers

The angles on the Unit Circle that make the equation -√3 csc(2θ) = 2 true are θ = π/6 + 2πn and θ = 5π/6 + 2πn, where n is an integer.

How to calculate the value

From the information, the following can be deduced:

-√3 csc(2θ) = 2

csc(2θ) = -2/√3

sin(2θ) = -√3/2

We know that sin(2θ) = 2sin(θ)cos(θ) by the double-angle identity for sine,

2sin(θ)cos(θ) = -√3/2

2sin(θ)cos(θ) = -√3/2

sin(θ)cos(θ) = -√3/4

sin(θ)cos(θ) = sin(π/3)sin(θ)

cos(θ) = sin(π/3)

θ = π/6 + 2πn, 5π/6 + 2πn

Therefore, the angles on the Unit Circle that make the equation -√3 csc(2θ) = 2 true are θ = π/6 + 2πn and θ = 5π/6 + 2πn, where n is an integer

Learn more about angle on

https://brainly.com/question/25716982

#SPJ1

A flagpole is 12 feet fall. Its shadow is
11 feet long. How far is it from the top of the flagpole to the end of its shadow?

Answers

The distance from the top of the flagpole to the end of its shadow is approximately 10.02 feet.

Explain the term distance

Distance refers to the measurement of the space between two objects or points. It is typically measured in units such as meters, kilometres, miles, or feet. The distance can be calculated using various methods, including using maps, GPS technology, or mathematical formulas.

According to the given information

We can set up the following proportion:

h / 12 = d / 11

We can cross-multiply to get:

h x 11 = 12 x d

Simplifying further:

d = (h x 11) / 12

We need to solve for d, so we need to find the value of h. Using the Pythagorean theorem, we can set up the following equation:

h² + d² = 12²

Substituting d = (h x 11) / 12, we get:

h² + ((h x 11) / 12)² = 12²

Simplifying:

h² + (121h²) / 144 = 144

Multiplying both sides by 144/265:

265h² / 144 = 144

Solving for h:

h² = (144 x 144) / 265

h = √(20736 / 265)

h = √(78.113)

Now we can substitute this value into our earlier equation to find d:

d = (√(78.113) x 11) / 12

d ≈ 10.02 feet

Therefore, the distance from the top of the flagpole to the end of its shadow is approximately 10.02 feet.

To know more about distance visit

brainly.com/question/15172156

#SPJ1

Andrea is playing a board game with her friends. A player spins the spinner shown below and receives the number of points indicated in the section where the arrow stops. A negative value means a loss of points.
What is the expected payoff, in points, for landing on a space of the board game?

Answers

The expected payoff for landing on a space of the board game is 2.67 points.

How to find the expected payoff?

We need to multiply each possible outcome by its probability of occurring and then add all the products to get the expected payoff.

Let's begin by determining the likelihood of each outcome:

The number 8 appears four times, so the probability of getting an 8 is 4/12 = 1/3.

The number 1 appears four times, so the probability of getting a 1 is also 1/3.

The number -2 appears twice, so the probability of getting a -2 is 2/12 = 1/6.

The number - 4 shows up two times, so the likelihood of getting a - 4 is likewise 1/6.

After that, we add up each outcome by multiplying it by its probability:

Expected payoff = (8 * 1/3) + (8 * 1/3) + (8 * 1/3) + (8 * 1/3) + (1 * 1/3) + (1 * 1/3) + (-2 * 1/6) + (-2 * 1/6) + (-4 * 1/6) + (-4 * 1/6)

Expected payoff = 2.67

As a result, the expected reward for landing on a board game space is 2.67 points.

know more about probability visit :

https://brainly.com/question/11234923

#SPJ1

pls!! :(( a golf ball has been hit off of the tee at an angle of elevation of 30 degrees and an initial velocity of 128 ft/sec

how long is the ball in the air (hang time)?

what is the maximum height of the ball?

how far, horizontally, does the ball travel in the air?

Answers

According to the information, the horizontal distance traveled by the ball is 443.404 feet.

How to calculate the distance traveled by the ball?

We can use the kinematic equations of motion to solve for the hang time, maximum height, and horizontal distance traveled by the golf ball.

First, we need to resolve the initial velocity vector into its horizontal and vertical components. The vertical component will determine the maximum height and hang time, while the horizontal component will determine the horizontal distance traveled.

The initial velocity can be represented as:

v0x = v0 cos(theta) = 128 cos(30) = 110.851 ft/secv0y = v0 sin(theta) = 128 sin(30) = 64 ft/sec

where v0 is the initial velocity, theta is the angle of elevation, v0x is the horizontal component of the initial velocity, and v0y is the vertical component of the initial velocity.

Now we can use the kinematic equations to solve for the hang time, maximum height, and horizontal distance traveled.

Hang time (time in air):

We can use the vertical motion equation to solve for the time when the ball reaches its maximum height:

v = v0y - gt0 = 64 - 32tt = 2 seconds

Since the ball will be in the air for twice the time it takes to reach its maximum height, the hang time is:

2t = 4 seconds

Maximum height:

We can use the vertical motion equation to solve for the maximum height reached by the ball:

y = v0y t - 1/2 gt^2y = 64(2) - 1/2 (32)(2)^2y = 64 ft

Therefore, the maximum height of the ball is 64 feet.

Horizontal distance traveled:

We can use the horizontal motion equation to solve for the horizontal distance traveled by the ball:

x = v0x t

x = 110.851(4)

x = 443.404 ft

Therefore, the horizontal distance traveled by the ball is 443.404 feet.

Learn more about horizontal distance in: https://brainly.com/question/10093142
#SPJ1

Two angles lie along a straight line. If m∠A is five times the sum of m∠B plus 7.2°, what is m∠B?

Answers

As a result, angle B has a 24 degree measure as the total of the two angles, which are along a straight line, is 180 degrees.

what is angle ?

Thus according their size or measurement, angles can be categorised. An oblique angle is larger than 90 degrees but far less than 180 degrees, a straight angle is exactly 90 degree, a right angle is turned 90 degrees, and an acute angle is less than 90 degrees. Reflex angles are angles that are higher than 180o but a little less than 360 degrees, and complete angles are angles that measure exactly 360 degrees. Geometry, trigonometry, physics, engineering, and many other branches of mathematics and science all make use of angles.

given

The total of the two angles, which are along a straight line, is 180 degrees. Let's refer to the angle B's measurement as x.

The information provided in the problem can then be used to create an equation as follows:

m∠A = 5(m∠B + 7.2°)

Due to the fact that the two angles are perpendicular to one another, we may replace mA with 180 - mB:

180 - m∠B = 5(m∠B + 7.2°)

The right side is being widened:

180 - m∠B = 5m∠B + 36

Simplifying and putting all the mB words to one side:

6m∠B = 144

m∠B = 24

As a result, angle B has a 24 degree measure as the total of the two angles, which are along a straight line, is 180 degrees.

To know more about angles visit:

https://brainly.com/question/14569348

#SPJ1

The table shows the length of the songs played by a radio station during a 90-minute period. Alicia is making a histogram of the data. What frequency should she show for the interval 160-169 seconds?

Answers

The frequency for the interval 160-169 seconds is 3.

What is mode of the data?

The value that appears the most frequently in a data set is its mode. In the data set, it is the number that appears the most frequently. For instance, in the subsequent data set:

2, 3, 4, 4, 5, 5, 5, 6, 7, 8

Since no other value appears more frequently, the mode is 5, which only appears three times. The data set is considered to have several modes if various values occur with the same greatest frequency. There is no mode if no value appears more than once in the data collection.

From the given table we see that the songs that have the time duration between 160 and 169 is:

162, 168, 165

Hence, the frequency for the interval 160-169 seconds is 3.

Learn more about mode here:

https://brainly.com/question/30891252

#SPJ1

The complete question is:

Where are the x-intercept(s) of the graph?

Answers

The x-intercept of the graph is (0,0).

What is an illustration of an x-intercept on a graph?

On a graph, the x-intercept is the point at which a line crosses the x-axis. At that time, the y coordinate has no value. The y-intercept is the point where the line crosses the y-axis. The x coordinate has no value. For example, y = x + 5 would intersect the x-axis at (-5, 0), forming the x-intercept.

From the figure, it is clear that the line crosses the X-axis at the origin, which means that x - coordinate 0 keeping y -coordinate is also zero.

Which means that the x-intercept of the graph is (0,0).

Learn more about intercept here:

https://brainly.com/question/14180189

#SPJ1

An item is worth $240 now. This is 30% of what it was originally worth. What was it originally worth?

Answers

To find out how much the item was originally worth, we will use the formula:
OP= P/1 - %
OP is the original price and P is the price now and % is the, well, percent. So let’s put the numbers in. Remember that you must convert the percent to a decimal by dividing it by 100.
OP = 240/1 - 0.30
OP = 240/0.7
OP= 342.8571428571429
OP (rounded) = $342.86
So the original price of the item was $342.86 before the 30% cut.

Hope this helped!

A 40" screen television at a popular electronics retailer is priced at $600. The wall mount for this sized television costs $29.99.

Part A: If Jamie purchases the television and the wall mount and has a coupon for 30% off, how much will her subtotal be? Show all necessary work. (4 points)

Part B: If Jamie makes the purchase in a state with a 7% state sales tax, what will her final total be? Show all necessary work. (2 points)

Part C: The electronics retailer is extending a special offer to install the wall mount and television for free. However, Jamie decides to tip the installation specialist 10% of the original purchase price before the discount is applied. How much would her new total be, including tax, discount, and tip? Show all necessary work. (4 points)

Answers

Jamie's subtotal will be $440.99Jamie's total will be $471.86Her new total  including tax, discount, and tip will be $531.86Cost estimation

The cost of the television and wall mount before discount is $600 + $29.99 = $629.99

After a 30% discount, the subtotal is:

$629.99 x 0.70 = $440.99

The sales tax is 7% of the subtotal:

$440.99 x 0.07 = $30.87

The final total is the subtotal plus the sales tax:

$440.99 + $30.87 = $471.86

The original purchase price before discount is $600.

10% of $600 is $60.

So Jamie decides to tip the installation specialist $60.

After the discount, the subtotal is $440.99 (as calculated in Part A).

The sales tax is 7% of the subtotal:

$440.99 x 0.07 = $30.87

The new total is the subtotal plus the sales tax and the tip:

$440.99 + $30.87 + $60 = $531.86

More on cost estimation can be found here: https://brainly.com/question/14347037

#SPJ1

determina el volumen cuyo diametro es de 8 y su altura de 15 cm

Answers

Answer:

3016 centímetros cúbicos.

Step-by-step explanation:

El radio del cilindro es de 8 cm y la altura es de 15 cm. Sustituya 8 por r y 15 por h en la fórmula . Simplifique. Por lo tanto, el volumen del cilindro es de alrededor de 3016 centímetros cúbicos.

A cow is tethered to one corner of a square barn, 8 feet by 8 feet, with a rope 130 feet long. What is the maximum grazing area for the cow?

Answers

The maximum grazing area for the cow is approximately 53,343.08 square feet.

How to Find the maximum Grazing Area?

The maximum grazing area for the cow can be found by imagining a circle with radius equal to the length of the rope (130 feet) centered at the corner of the barn where the cow is tethered. The grazing area is the portion of the circle that lies outside the barn.

Since the barn is 8 feet by 8 feet, it covers a square area of 64 square feet. The radius of the circle is 130 feet, so the area of the circle is π(130)^2 square feet.

To find the maximum grazing area, we need to subtract the area of the barn from the area of the circle.

Area of circle = π(130)^2 square feet = 53,407.08 square feet

Area of barn = 64 square feet

Maximum grazing area = Area of circle - Area of barn

= 53,407.08 - 64

= 53,343.08 square feet (rounded to two decimal places)

Learn more about maximum grazing area on:

https://brainly.com/question/31253278

#SPJ1

Use the formula KE= mv^2/2 where m=mass, V= velocity, KE = kinetic energy. If dev has a mass of 60kg and is running at a constant velocity with 150 J of KE. What is his velocity?

Answers

Dev's velocity is [tex]\sqrt{5}[/tex]. Thus option B.

What is kinetic energy?

Kinetic energy is a amount of energy possessed when an object is in motion. Such that;

KE = 1/2 m v^2

Where m = mass, v = velocity

It is measured in Joules.

From the given question, we have;

KE = 1/2 m v^2

2KE = m v^2

v^2 = 2KE/ m

      = (2*150)/ 60

      = 300/ 60

      = 5

V = (5)^1/2

The velocity of Dev is B. [tex]\sqrt{5}[/tex].

Learn more about kinetic energy at https://brainly.com/question/25959744

#SPJ1

if you multiplied a number by 1/2 , the result would be Responses

Answers

Answer:

half the number you started with

Step-by-step explanation:

8 times 1/2 would be 4....6 times 1/2 would be 3!

It would be like divided a number by 2.

4 divide by 3/5 as a fration

Answers

Answer:

6 and 2/3

Step-by-step explanation:

4 divided by 3/5 is the same as 4 divided by 0.6

4 divided by 0.6 equals 6.6 repeating...

or

6 and 2/3

What 4×4/3 in its simplest form

Answers

(4)(4/3)
To start, we will change the whole number 4 into a fraction. Because when multiplying fractions, you will multiply the numerator and denominator across. So 4 = 4/1.
(4/1)(4/3)
Now multiply the numerators and denominators across:
16/3
That’s your answer in improper fraction form, or 5 1/3 in mixed number, or 5.333… in decimal form.

Hope this helped!

Answer:

5 [tex]\frac{1}{3}[/tex]

Step-by-step explanation:

[tex]\frac{4}{1}[/tex] x [tex]\frac{4}{3}[/tex] = [tex]\frac{16}{3}[/tex]

You can re-write [tex]\frac{16}{3}[/tex] as

[tex]\frac{3}{3}[/tex] + [tex]\frac{3}{3}[/tex] + [tex]\frac{3}{3}[/tex] + [tex]\frac{3}{3}[/tex] + [tex]\frac{3}{3}[/tex] + [tex]\frac{1}{3}[/tex]  I wrote it like this because every [tex]\frac{3}{3}[/tex] is equal to 1.

1 + 1 + 1 + 1 + 1 + [tex]\frac{1}{3}[/tex] = 5[tex]\frac{1}{3}[/tex]

Helping in the name of Jesus.

if k(x) = 3x, then f'(x)=? A. x³Ln3 B. 3xLn3 C. 3x/Lnx D. 3/3xLn3​

Answers

The correct option is B .solution of given problem with the help of integrating the given function is 3xLn3

what is integration and function ?

The area under a curve in a given range can be calculated mathematically via integration. To locate the region between the curve and the x-axis, it is necessary to find a function's antiderivative and evaluate it twice.

A function is a rule that gives each input value a distinct output value. It can be compared to a machine that processes inputs into outputs in accordance with a predetermined rule or formula.

According to given information

To find f'(x), we need to take the derivative of f(x), where f(x) is the antiderivative of k(x).

Since k(x) = 3x, we can find f(x) by integrating 3x with respect to x:

f(x) = ∫ 3x dx = 3/2 x² + C

where C is a constant of integration.

Now we can find f'(x) by taking the derivative of f(x):

f'(x) = d/dx (3/2 x² + C) = 3x

Therefore, the answer is (B) 3xLn3. Option (A) is incorrect because there is no natural logarithm term in the derivative of f(x). Option (C) is incorrect because the derivative of 3x is 3, not 3/Ln(x). Option (D) is incorrect because there is no x in the denominator of the natural logarithm term.

To know more about integration Visit:

brainly.com/question/31433890

#SPJ1

please help pleaseee i need dis good grade

Answers

Answer:62.8 units

Step-by-step explanation:

Other Questions
Keynesian and Classical economists agree on how the economy works in the long-run. True or False? Briefly explain the reason why the statement is true or false. The wheels on a bike have a diameter of 26 inches. How many full revolutions will the wheels need to make to travel 100 feet? how did the frequency of stimulation effect of force generated by the isolated skeletal muscle when the as a rule of thumb, what is the minimum time needed to change a professional reputation Project STAR: does classroom size have an effect on learning in children? 3 different groups: Is a triangle with sides of 5 meters, 12 meters, and 13 meters a right triangle? Explain/Show your work. Dynamo Electronics, a major maker of electronics products in the United Kingdom, is concerned about the continually falling prices of the electronics products it makes due to greater competition abroad. To offset some of its business risk, it decides to merge with a major business consulting firm in the United States, IRM. What type of merger is this? If the mitochondrial matrix is impermeable to NADH, how do they cross? Fiona invents a new deep sea fishing net, which she names "Great Catch. She also write the operating manual to be included with each net. Fiona could obtain copyright protection for a. the manual only b. the name only c. the net only d. the manual, the net, and the name Write the correct ionic formula for the compound formed between the following:A. Na and O B. Br and Al help me ASAP I really need help :( Group 6: Assume you are group responsible for customer service in a dairy company in different area. Present your effective presence of your groups and what is your responsibilities especially with online grocery? HURRY DUE TONIGHTJenny mixed 4 kg of mixed nuts containing 16% peanuts with 12 kg of mixed nuts containing 40% peanuts. What percent of the new mixture if peanuts?A. 34%B. 13% Fred's science class is going to the Ancient Dig adventure park. His teacher got the school group package, which costs $308. The package covers the entrance fee for the group, a private dig site, and an activity guide. Fred's teacher upgraded the package to include a fossil tool kit for each of the 22 students, bringing the total to $440. Which equation can you use to find k, the cost of each tool kit? this genre many times has an unsympathetic anti-hero T/f according to bourdieu symbolic power involves the power of creating or naming the world through words please help asap this is for geometry its R.4 Please help !!i only need help with the second problem which is:5. For 0x 6, express g(x) in terms of x. Do not include +C in your final answer.Your help is very appreciated !! Question 14 Marks: 1 Which type of inspection would be conducted prior to legal action?Choose one answer. a. reinspection b. routine c. complete d. initial A pay system in which compensation is based on the number of units produced rather than the number of hours worked is